Understanding the Multivariate Limit of Sin(xy)/x

Click For Summary
SUMMARY

The limit of Sin(xy)/x as (x,y) approaches (0,y) simplifies to y. This conclusion is reached by applying L'Hospital's rule, treating y as a constant during the differentiation process. The behavior of Sin(θ) for small values of θ also supports this result, confirming that the limit can be effectively evaluated using these mathematical techniques.

PREREQUISITES
  • Understanding of multivariable limits
  • Familiarity with L'Hospital's rule
  • Knowledge of the behavior of the sine function for small angles
  • Basic calculus concepts
NEXT STEPS
  • Study the application of L'Hospital's rule in multivariable calculus
  • Explore the Taylor series expansion of the sine function
  • Investigate other techniques for evaluating limits in multivariable contexts
  • Learn about continuity and differentiability in multivariable functions
USEFUL FOR

Students and educators in calculus, mathematicians focusing on limits, and anyone seeking to deepen their understanding of multivariable calculus concepts.

trap101
Messages
339
Reaction score
0
So I figured out the solution to lim (x,y)-->(0,y) of Sin(xy)/x, but I figured it out by looking at a solution. I wanted to understand though why with respect to the above limit how Sin(xy)/x = y?

How do you separate the xy in the numerator?
 
Physics news on Phys.org
trap101 said:
So I figured out the solution to lim (x,y)-->(0,y) of Sin(xy)/x, but I figured it out by looking at a solution. I wanted to understand though why with respect to the above limit how Sin(xy)/x = y?

How do you separate the xy in the numerator?

Use l'Hospitals's rule, or else look at the behavior of ##\sin \theta## for small ##|\theta|##.
 
Ray Vickson said:
Use l'Hospitals's rule, or else look at the behavior of ##\sin \theta## for small ##|\theta|##.


Using L'Hospital's rule, would I essentially be treating "y" as a constant?
 
trap101 said:
Using L'Hospital's rule, would I essentially be treating "y" as a constant?

Yes!
 
Question: A clock's minute hand has length 4 and its hour hand has length 3. What is the distance between the tips at the moment when it is increasing most rapidly?(Putnam Exam Question) Answer: Making assumption that both the hands moves at constant angular velocities, the answer is ## \sqrt{7} .## But don't you think this assumption is somewhat doubtful and wrong?

Similar threads

  • · Replies 6 ·
Replies
6
Views
2K
  • · Replies 2 ·
Replies
2
Views
2K
  • · Replies 8 ·
Replies
8
Views
2K
Replies
5
Views
2K
  • · Replies 2 ·
Replies
2
Views
2K
  • · Replies 2 ·
Replies
2
Views
1K
Replies
13
Views
2K
Replies
5
Views
1K
Replies
11
Views
2K
  • · Replies 8 ·
Replies
8
Views
1K